Mathcenter Forum

Mathcenter Forum (https://www.mathcenter.net/forum/index.php)
-   ข้อสอบโอลิมปิก (https://www.mathcenter.net/forum/forumdisplay.php?f=28)
-   -   ช่วยเฉลยหน่อยครับ (https://www.mathcenter.net/forum/showthread.php?t=10580)

Siren-Of-Step 13 เมษายน 2010 15:49

ช่วยเฉลยหน่อยครับ
 
บางข้อ ก็ได้แล้ว บางข้อก็ไม่ได้ ช่วยเฉลยหน่อยครับ

จาก พี่ Innoxent
1. มีชุดอันดับ $(a,b,c,d)$ อยู่ทั้งหมดกี่ชุด โดยที่ $a,b,c,d \in I^{+}$ ที่มีค่าไม่เกิน12 และ $a+2b+3c+4d$ หารด้วย 12 ลงตัว
ก. 648 ชุด ข. 960 ชุด ค. 1320 ชุด ง. 1728 ชุด

2. จงหาเศษเหลือจากการหาร $16^{16^{1}} + 16^{16^{2}} + 16^{16^{3}} + ... + 16^{16^{2551}}$ ด้วย 13
ก. 2 ข. 3 ค. 8 ง. 9

3. ให้ ABC เป็นสามเหลี่ยมที่ AB = 9 และ AC = 7 ให้ D เป็นจุดกึ่งกลางของด้าน BC ให้ X เป็นจุดบนด้าน AB ซึ่ง DX ตั้งฉากกับ AB ให้ Y เป็นจุดบนด้าน AC ซึ่ง DY ตั้งฉากกับ AC ถ้า AX = 6 จงหาความยาวของ AY
ก. $\frac{34}{7}$ ข. $\frac{36}{7}$ ค. $\frac{38}{7}$ ง. $\frac{40}{7}$

4. กำหนด $H_n = \frac{1}{1} + \frac{1}{2} + \frac{1}{3} + ... \frac{1}{n}$ ถ้าทราบว่า $7.4854 < H_{1000} < 7.4855$ จงหาจำนวนเต็มที่มากที่สุด ที่มีค่าน้อยกว่า $H_1 + H_2 + H_3 + ... + H_{1000}$
ก. 6490 ข. 6492 ค. 6494 ง. 6496

5. จงหาชุดของจำนวนเฉพาะ $(p,q,r,s)$ ทั้งหมดที่ $p \geqslant q \geqslant r \geqslant s$ และ $pqrs = p + q + r +s + 1834$

6. ให้ a และ b เป็นจำนวนจริง ซึ่ง $(2a+b)^2 + (a+\frac{1}{3})^2 + (b + \frac{4}{3})^2 = \frac{2}{3}$
จงหาค่าของ $(2b+a)^2 + (b+\frac{1}{3})^2 + (a+\frac{4}{3})^2$

7. จงหาเศษเหลือจากการหาร $1053^3 + 392^3 + 378^3$ ด้วย 2579

8. ABC เป็นสามเหลี่ยมด้านเท่า ให้ P เป็นจุดที่อยู่บนด้าน AC โดยที่ $P\hat B C = 23^\circ$ และต่อรังสี BP ไปจนถึงจุด Q โดยที่ $P\hat C Q = 67^\circ$ จงหาขนาดของมุม $A\hat Q P$

9. จงหาคู่อันดับของจำนวนนับ (n,k) ทั้งหมดที่ทำให้ $\frac{2(kn-1)}{n+1}$ และ $\frac{6kn-1}{2k-1}$
เป็นจำนวนเต็มทั้งสองจำนวน

10. ให้ ABCD เป็นสี่เหลี่ยมที่ $AB = \sqrt{5}$ , $BC = \sqrt{13}$ , $CD = \sqrt{17}$ ,$DA = \sqrt{29}$ ,$AC = \sqrt{10}$ จงหาพื้นที่ของสี่เหลี่ยม ABCD

11. มีจำนวนเต็มบวก n ทั้งหมดกี่จำนวน ที่ $1 \leqslant n \leqslant 1,000,000$ และ $n^n$ หารด้วย $n^2+1$ เหลือเศษเป็นกำลังสองสมบูรณ์

12. ให้ $A= \frac{1}{\sqrt[3]{1,000}} + \frac{1}{\sqrt[3]{1,001}} + \frac{1}{\sqrt[3]{1,002}} + ...\frac{1}{\sqrt[3]{1,000,000}}$ จงหาจำนวนเต็มที่มากที่สุดที่มีค่าน้อยกว่า $\frac{A}{4}$

13. ให้ a,b,c เป็นจำนวนจริงใดๆ ที่แตกต่างกัน ซึ่ง $ab+ba+ca > 0$ จงหาค่าต่ำสุดที่เป็นไปได้ ของ $\frac{(a^2+b^2+c^2)^4}{(ab+bc+ca)(a-b)^2(b-c)^2(c-a)^2}$

14. ให้ ABC เป็นสามเหลี่ยมที่ $B\hat A C = 73^\circ$ และ $A\hat B C > B\hat C A$ ให้ P เป็นจุดบนด้าน AC ที่ทำให้ $P\hat B C = B\hat C A$ และ ให้ Q เป็นจุดบนด้าน AB โดยที่ BP และ CQ ตัดกันที่ X ถ้า $CX = AP+PX$ จงหาขนาดของมุม $B\hat X Q$

15. ให้ x,y,z เป็นจำนวนจริงใดๆ
จงหาค่าต่ำสุดที่เป็นไปได้ของ $\frac{x^2}{(4x-3y-z)^2} + \frac{y^2}{(4y-3z-x)^2} + \frac{z^2}{(4z-3x-y)^2}$

16. จงหาจำนวนนับ n ที่มากที่สุด ที่มีสมบัติว่า เราสามารถแบ่งจำนวนนับตั้งแต่ 1 ถึง n ออกเป็นสองกลุ่มได้ โดยที่ผลบวกของจำนวนสองจำนวนใดๆ ที่อยู่ในกลุ่มเดียวกัน จะต้องไม่เป็นกำลังสามสมบูรณ์

โอลิมปิก 44

17. $f(x) = ax^4 + bx^3 + cx^2$ เมื่อ a,b และ c เป็นค่าคงตัวที่ทำให้
$f(x(x+1)) - f(x(x-1)) = x^7$ ถ้า $1^7 + 2^7 + 3^7 + .. + n^7 = \frac{n^2(n+1)^2}{24} \bullet p(n)$ แล้ว $p(n)$ เป็นพหุนามใด

18. กำหนดให้ n เป็นจำนวนเต็มบวกที่มากที่สุด ซึ่ง $n-6$ หาร $n^3+n^2+11$ ลงตัว ถ้า $p > 1$ เป็นจำนวนเต็มบวกซึ่งหาร n ลงตัวแล้ว p คือจำนวนใด

19. กำหนดให้ A เป็นจำนวนเต็มบวกสี่หลัก ถ้าผลบวกของเลขโดดทั้งหมดของ A เท่ากับจำนวนซึ่งตัดสองหลักหลังของ A ออกไป และผลคูณของเลขโดดทั้งหมดของ A เท่ากับ จำนวนซึ่งเท่ากับจำนวนซึ่งตัดสองหลักแรกของ A ออกไป แล้ว A คือจำนวนใด

20. กำหนดให้ B เป็นจำนวนเต็มบวก ซึ่งมีเลข 6 ในหลักหน่วย เมื่อเอา 6 ในหลักหน่วย ย้ายไปไว้ในหลักแรก ทำให้ได้จำนวนใหม่ ซึ่งมีค่าเป็น 4 เท่าของ B จงหาค่า B ที่น้อยที่สุด

21. abc เป็นจำนวนเต็มสามหลัก ซึ่งมี a, b และ c เป็นเลขโดด
ถ้า $ abc = a! + b! + c!$ จงหา $a+b+c$

22. กำหนดให้$\alpha$ และ $\beta$ เป็นรากของสมการ $ax^2 + bx + c = 0$ เมื่อ a,b และ c เป็นค่าคงตัว ซึ่ง $a \not= b$ ถ้า $\beta = \alpha^2$ แล้ว a ,b และ c สัมพันธ์กันอย่างไร

23. กำหนดให้ $ax+b$ เป็นเศษเหลือของสมการ
$2x - 2^2x^2 + 2^3x^3 - 2^4x^4 + ... + (-1)^{n+1}2^nx^n$ ด้วย $4x^2-1$ ถ้า n = 100 แล้ว จงหาค่า a และ b

C H O 19 เมษายน 2010 20:20

พิจารณาเลขชี้กำลังของแต่ละพจน์ (คือ $16^1,16^2,...,16^{2551}$)
$16^1\equiv 1 (mod 3)$
$16^n\equiv 1^n\equiv 1 (mod 3)$
ดังนั้น เลขชี้กำลังของแต่ละพจนื หารด้วย 3 เหลือเศษ 1 เสมอ

พิจารณา
$16^1\equiv 3 (mod 13)$
$16^2\equiv 3^2 \equiv 9 (mod 13)$
$16^3\equiv 9\cdot 16 \equiv 144 \equiv 1 (mod 13)$
$16^4=16^1\cdot 16^3 \equiv 3 (mod 13)$
$16^7=16^1\cdot 16^4 \equiv 3 (mod 13)$
เนื่องจากแต่ละพจน์อยู่ในรูป 16 ยกกำลังเลขที่หารด้วย 3 แล้วเหลือเศษ 1
ดังนั้น ทุกพจน์สมภาคกับ 3 (mod 13)
ดังนั้น คำตอบ $\equiv 2551\cdot 3 (mod 13) \equiv 7653 \equiv 9 (mod 13)$
ตอบ 9 ครับ :)

กระบี่เดียวดายแสวงพ่าย 20 เมษายน 2010 11:01

ข้อ23.นะครับ จากทฤษฎีเศษเหลือ จะได้ 4x^2-1=0 แล้วเทอม X^2, x^4 ,.... จะหาค่าได้เป็น1/4 , 1/16 , .... ตามลำดับ
ส่วนเทอม x^3 , x^5 , ... จะกลายเป็น x/4, x/16, ... ตามลำดับ และจะได้ 2x-1+ 2x-1+.... + 2x-1 มี เทอม 2x-1 อยู่ 50 เทอม
เศษเหลือคือ 100x-50 ครับ ดังนั้น a=100 , b=-50

Toru 20 เมษายน 2010 15:02

ข้อ 1.เพราะว่า {-1,-2,-3,...,-12} เป็น complete residue system modulo 12

พิจารณา a+2b+3c+4d ≡ (mod 12)นั่นคือ 2b+3c+4d ≡ -a(mod12)

เพราะว่า -a∈{-1,-2,-3,...,-12} จะได้ว่า ทุก 2b+3c+4d ที่เลือกมา จะได้ว่ามี -a ตัวเดียวที่ทำให้2b+3c+4d≡-a(mod12)

แสดงว่าเราสามารถเลือก b c d ได้อย่างอิสระ จะทำให้ได้ a เพียงตัวเดียวเสมอ ที่ทำให้สมการเป็นจริง
ฉะนั้น (a,b,c,d) ที่เป็นไปได้ทั้งหมดมี 1x12x12x12=1,728 ชุด ครับ

C H O 20 เมษายน 2010 20:38

ข้อ 18.
$n^3+n^2+11=(n-6)(n^2+7n+42)+263$
ดังนั้น $\frac{n^3+n^2+11}{n-6}=n^2+7n+42+\frac{263}{n-6}$
ซึ่งจำนวนเต็ม n ที่มากที่สุด ที่ทำให้ทางขวามือเป็นจำนวนเต็ม คือ ค่า $n$ ที่ n-6=263 ซึ่งคือ n=269
เนื่องจาก n=269 เป็นจำนวนเฉพาะ ดังนั้น p=269 :)

กระบี่เดียวดายแสวงพ่าย 20 เมษายน 2010 21:13

ข้อ 21 นะครับ 145 = 1!+4!+5! คำตอบคือ10 ครับ

PP_nine 01 พฤษภาคม 2010 10:28

อ้างอิง:

ข้อความเดิมเขียนโดยคุณ กระบี่เดียวดายแสวงพ่าย (ข้อความที่ 85533)
ข้อ 21 นะครับ 145 = 1!+4!+5! คำตอบคือ10 ครับ

ลองดูเว็บนี้ครับ :)


เวลาที่แสดงทั้งหมด เป็นเวลาที่ประเทศไทย (GMT +7) ขณะนี้เป็นเวลา 06:29

Powered by vBulletin® Copyright ©2000 - 2024, Jelsoft Enterprises Ltd.
Modified by Jetsada Karnpracha